Bifurcation in a linear system with 2 equations and 1parameter












1














I have the following system $$frac{dx}{dt} = ax+y$$ and $$frac{dy}{dt} = -x+ay$$ By setting the derivative equals to $0$, we get the equilibrium point $(0,0)$. After drawing the phase portrait for various parameter $a$, we conclude that $(0,0)$ is stable when $a<0$ (see 1), a center when $a=0$ (see 2), and unstable when $a>0$ (see 3). Also, the eigenvalues are $lambda = a pm i$.



Do we have a bifurcation here? I was guessing that maybe we have Hopf bifurcation when $a=0$ because $(0,0)$ lose its stability when $a=0$. If it's true, I was also asked to draw the bifurcation diagram. I am not sure what it means or how to draw it.










share|cite|improve this question




















  • 3




    We do have a bifurcation here since equilibrium's stability has changed. However, it's not a Hopf bufurcation: since the system is linear for all values of $a$, there is no limit cycle (isolated closed trajectory). Also, typically at critical value for Hopf bifurcation we have a degenerate focus, not a center as here. I think you can take a look at simplest bifurcation diagrams in Strogatz's book.
    – Evgeny
    2 days ago












  • @Evgeny was the book you refer to is Nonlinear Dynamics And Chaos: With Applications To Physics, Biology, Chemistry, And Engineering by Steven H. Strogatz? and just to clarify, so Hopf bifurcation could only occur when the system is nonlinear?
    – aaaaaa
    yesterday






  • 2




    Yeah, that was exactly the book I was referring to. And yes, Hopf bifurcation (changing stability of equilibrium by emerging/colliding with limit cycle) can happen only when system is nonlinear: linear systems don't have limit cycles.
    – Evgeny
    yesterday
















1














I have the following system $$frac{dx}{dt} = ax+y$$ and $$frac{dy}{dt} = -x+ay$$ By setting the derivative equals to $0$, we get the equilibrium point $(0,0)$. After drawing the phase portrait for various parameter $a$, we conclude that $(0,0)$ is stable when $a<0$ (see 1), a center when $a=0$ (see 2), and unstable when $a>0$ (see 3). Also, the eigenvalues are $lambda = a pm i$.



Do we have a bifurcation here? I was guessing that maybe we have Hopf bifurcation when $a=0$ because $(0,0)$ lose its stability when $a=0$. If it's true, I was also asked to draw the bifurcation diagram. I am not sure what it means or how to draw it.










share|cite|improve this question




















  • 3




    We do have a bifurcation here since equilibrium's stability has changed. However, it's not a Hopf bufurcation: since the system is linear for all values of $a$, there is no limit cycle (isolated closed trajectory). Also, typically at critical value for Hopf bifurcation we have a degenerate focus, not a center as here. I think you can take a look at simplest bifurcation diagrams in Strogatz's book.
    – Evgeny
    2 days ago












  • @Evgeny was the book you refer to is Nonlinear Dynamics And Chaos: With Applications To Physics, Biology, Chemistry, And Engineering by Steven H. Strogatz? and just to clarify, so Hopf bifurcation could only occur when the system is nonlinear?
    – aaaaaa
    yesterday






  • 2




    Yeah, that was exactly the book I was referring to. And yes, Hopf bifurcation (changing stability of equilibrium by emerging/colliding with limit cycle) can happen only when system is nonlinear: linear systems don't have limit cycles.
    – Evgeny
    yesterday














1












1








1







I have the following system $$frac{dx}{dt} = ax+y$$ and $$frac{dy}{dt} = -x+ay$$ By setting the derivative equals to $0$, we get the equilibrium point $(0,0)$. After drawing the phase portrait for various parameter $a$, we conclude that $(0,0)$ is stable when $a<0$ (see 1), a center when $a=0$ (see 2), and unstable when $a>0$ (see 3). Also, the eigenvalues are $lambda = a pm i$.



Do we have a bifurcation here? I was guessing that maybe we have Hopf bifurcation when $a=0$ because $(0,0)$ lose its stability when $a=0$. If it's true, I was also asked to draw the bifurcation diagram. I am not sure what it means or how to draw it.










share|cite|improve this question















I have the following system $$frac{dx}{dt} = ax+y$$ and $$frac{dy}{dt} = -x+ay$$ By setting the derivative equals to $0$, we get the equilibrium point $(0,0)$. After drawing the phase portrait for various parameter $a$, we conclude that $(0,0)$ is stable when $a<0$ (see 1), a center when $a=0$ (see 2), and unstable when $a>0$ (see 3). Also, the eigenvalues are $lambda = a pm i$.



Do we have a bifurcation here? I was guessing that maybe we have Hopf bifurcation when $a=0$ because $(0,0)$ lose its stability when $a=0$. If it's true, I was also asked to draw the bifurcation diagram. I am not sure what it means or how to draw it.







differential-equations dynamical-systems mathematical-modeling stability-in-odes bifurcation






share|cite|improve this question















share|cite|improve this question













share|cite|improve this question




share|cite|improve this question








edited 16 hours ago

























asked 2 days ago









aaaaaa

407




407








  • 3




    We do have a bifurcation here since equilibrium's stability has changed. However, it's not a Hopf bufurcation: since the system is linear for all values of $a$, there is no limit cycle (isolated closed trajectory). Also, typically at critical value for Hopf bifurcation we have a degenerate focus, not a center as here. I think you can take a look at simplest bifurcation diagrams in Strogatz's book.
    – Evgeny
    2 days ago












  • @Evgeny was the book you refer to is Nonlinear Dynamics And Chaos: With Applications To Physics, Biology, Chemistry, And Engineering by Steven H. Strogatz? and just to clarify, so Hopf bifurcation could only occur when the system is nonlinear?
    – aaaaaa
    yesterday






  • 2




    Yeah, that was exactly the book I was referring to. And yes, Hopf bifurcation (changing stability of equilibrium by emerging/colliding with limit cycle) can happen only when system is nonlinear: linear systems don't have limit cycles.
    – Evgeny
    yesterday














  • 3




    We do have a bifurcation here since equilibrium's stability has changed. However, it's not a Hopf bufurcation: since the system is linear for all values of $a$, there is no limit cycle (isolated closed trajectory). Also, typically at critical value for Hopf bifurcation we have a degenerate focus, not a center as here. I think you can take a look at simplest bifurcation diagrams in Strogatz's book.
    – Evgeny
    2 days ago












  • @Evgeny was the book you refer to is Nonlinear Dynamics And Chaos: With Applications To Physics, Biology, Chemistry, And Engineering by Steven H. Strogatz? and just to clarify, so Hopf bifurcation could only occur when the system is nonlinear?
    – aaaaaa
    yesterday






  • 2




    Yeah, that was exactly the book I was referring to. And yes, Hopf bifurcation (changing stability of equilibrium by emerging/colliding with limit cycle) can happen only when system is nonlinear: linear systems don't have limit cycles.
    – Evgeny
    yesterday








3




3




We do have a bifurcation here since equilibrium's stability has changed. However, it's not a Hopf bufurcation: since the system is linear for all values of $a$, there is no limit cycle (isolated closed trajectory). Also, typically at critical value for Hopf bifurcation we have a degenerate focus, not a center as here. I think you can take a look at simplest bifurcation diagrams in Strogatz's book.
– Evgeny
2 days ago






We do have a bifurcation here since equilibrium's stability has changed. However, it's not a Hopf bufurcation: since the system is linear for all values of $a$, there is no limit cycle (isolated closed trajectory). Also, typically at critical value for Hopf bifurcation we have a degenerate focus, not a center as here. I think you can take a look at simplest bifurcation diagrams in Strogatz's book.
– Evgeny
2 days ago














@Evgeny was the book you refer to is Nonlinear Dynamics And Chaos: With Applications To Physics, Biology, Chemistry, And Engineering by Steven H. Strogatz? and just to clarify, so Hopf bifurcation could only occur when the system is nonlinear?
– aaaaaa
yesterday




@Evgeny was the book you refer to is Nonlinear Dynamics And Chaos: With Applications To Physics, Biology, Chemistry, And Engineering by Steven H. Strogatz? and just to clarify, so Hopf bifurcation could only occur when the system is nonlinear?
– aaaaaa
yesterday




2




2




Yeah, that was exactly the book I was referring to. And yes, Hopf bifurcation (changing stability of equilibrium by emerging/colliding with limit cycle) can happen only when system is nonlinear: linear systems don't have limit cycles.
– Evgeny
yesterday




Yeah, that was exactly the book I was referring to. And yes, Hopf bifurcation (changing stability of equilibrium by emerging/colliding with limit cycle) can happen only when system is nonlinear: linear systems don't have limit cycles.
– Evgeny
yesterday















active

oldest

votes











Your Answer





StackExchange.ifUsing("editor", function () {
return StackExchange.using("mathjaxEditing", function () {
StackExchange.MarkdownEditor.creationCallbacks.add(function (editor, postfix) {
StackExchange.mathjaxEditing.prepareWmdForMathJax(editor, postfix, [["$", "$"], ["\\(","\\)"]]);
});
});
}, "mathjax-editing");

StackExchange.ready(function() {
var channelOptions = {
tags: "".split(" "),
id: "69"
};
initTagRenderer("".split(" "), "".split(" "), channelOptions);

StackExchange.using("externalEditor", function() {
// Have to fire editor after snippets, if snippets enabled
if (StackExchange.settings.snippets.snippetsEnabled) {
StackExchange.using("snippets", function() {
createEditor();
});
}
else {
createEditor();
}
});

function createEditor() {
StackExchange.prepareEditor({
heartbeatType: 'answer',
autoActivateHeartbeat: false,
convertImagesToLinks: true,
noModals: true,
showLowRepImageUploadWarning: true,
reputationToPostImages: 10,
bindNavPrevention: true,
postfix: "",
imageUploader: {
brandingHtml: "Powered by u003ca class="icon-imgur-white" href="https://imgur.com/"u003eu003c/au003e",
contentPolicyHtml: "User contributions licensed under u003ca href="https://creativecommons.org/licenses/by-sa/3.0/"u003ecc by-sa 3.0 with attribution requiredu003c/au003e u003ca href="https://stackoverflow.com/legal/content-policy"u003e(content policy)u003c/au003e",
allowUrls: true
},
noCode: true, onDemand: true,
discardSelector: ".discard-answer"
,immediatelyShowMarkdownHelp:true
});


}
});














draft saved

draft discarded


















StackExchange.ready(
function () {
StackExchange.openid.initPostLogin('.new-post-login', 'https%3a%2f%2fmath.stackexchange.com%2fquestions%2f3052293%2fbifurcation-in-a-linear-system-with-2-equations-and-1parameter%23new-answer', 'question_page');
}
);

Post as a guest















Required, but never shown






























active

oldest

votes













active

oldest

votes









active

oldest

votes






active

oldest

votes
















draft saved

draft discarded




















































Thanks for contributing an answer to Mathematics Stack Exchange!


  • Please be sure to answer the question. Provide details and share your research!

But avoid



  • Asking for help, clarification, or responding to other answers.

  • Making statements based on opinion; back them up with references or personal experience.


Use MathJax to format equations. MathJax reference.


To learn more, see our tips on writing great answers.





Some of your past answers have not been well-received, and you're in danger of being blocked from answering.


Please pay close attention to the following guidance:


  • Please be sure to answer the question. Provide details and share your research!

But avoid



  • Asking for help, clarification, or responding to other answers.

  • Making statements based on opinion; back them up with references or personal experience.


To learn more, see our tips on writing great answers.




draft saved


draft discarded














StackExchange.ready(
function () {
StackExchange.openid.initPostLogin('.new-post-login', 'https%3a%2f%2fmath.stackexchange.com%2fquestions%2f3052293%2fbifurcation-in-a-linear-system-with-2-equations-and-1parameter%23new-answer', 'question_page');
}
);

Post as a guest















Required, but never shown





















































Required, but never shown














Required, but never shown












Required, but never shown







Required, but never shown

































Required, but never shown














Required, but never shown












Required, but never shown







Required, but never shown







Popular posts from this blog

Human spaceflight

Can not write log (Is /dev/pts mounted?) - openpty in Ubuntu-on-Windows?

File:DeusFollowingSea.jpg